a我考网

 找回密码
 立即注册

QQ登录

只需一步,快速开始

扫一扫,访问微社区

查看: 137|回复: 0

[考试辅导] GMAT考试写作例文224篇连载(六一)

[复制链接]
发表于 2012-8-15 21:50:16 | 显示全部楼层 |阅读模式
61. The following appeared in the letters-to-the-editor section of a local newspaper.7 J0 A  D# u) j9 c
“Muscle Monthly, a fitness magazine that regularly features pictures of bodybuilders using state-of-the-art exercise machines, frequently sells out, according to the owner of Skyview Newsstand. To help maximize fitness levels in our town’s residents, we should, therefore, equip our new community fitness center with such machines.”
! c' l2 I- Y- RDiscuss how well reasoned... etc.
2 ~- z1 @8 u8 ^! H/ X1 z# ~& [0 |  }6 m6 {
In this argument the author concludes that the new community fitness center should be equipped with the state-of-the-art exercise machines featured in Muscle Monthly magazine. In support of this recommendation two reasons are offered: (1) Muscle Monthly contains pictures of bodybuilders using such machines, and (2) Muscle Monthly is a popular magazine, as evidenced by the fact that it frequently sells out at the local newsstand. This argument is questionable on two counts., Q/ X5 Z) A1 Q1 E  I: u! U2 s/ w
First, a major implication of the argument is that the bodybuilders pictured using the machines in Muscle Monthly magazine reached their state of fitness as a result of using these machines. The only evidence offered to support this contention, however, is the pictures in the magazine. It is possible that the bodybuilders pictured use different equipment for their workouts and are merely posing with the machines for advertising purposes.
2 w: {  ~  Z) o. z! HSecond, the author assumes that machines that are suitable for bodybuilding will also be suitable to help maximize the fitness levels of the town’s residents. This assumption is highly questionable. Machines designed to increase muscle development are significantly different from those designed to increase cardiovascular fitness. Consequently, it is unlikely that the machines pictured in the magazine will be of much use to help maximize the fitness levels of the town’s residents.
, e, _/ X3 ]0 a: `$ S6 W& {  ?5 dIn conclusion, this argument is unconvincing. To strengthen the argument the author would have to show that the bodybuilders pictured using the exercise machines actually used the machines to reach their level of muscle development. Additionally, the author would have to show that the machines were suitable for increasing the fitness levels of the persons using them.
回复

使用道具 举报

您需要登录后才可以回帖 登录 | 立即注册

本版积分规则

Archiver|手机版|小黑屋|Woexam.Com ( 湘ICP备18023104号 )

GMT+8, 2024-5-9 05:31 , Processed in 0.166362 second(s), 21 queries .

Powered by Discuz! X3.4 Licensed

© 2001-2017 Comsenz Inc.

快速回复 返回顶部 返回列表